Could I get a break down on this one please?
Could you guys break this one down for me please?
masonolinde on February 8, 2022
  • October 2005 LSAT
  • SEC3
  • Q22
3
Replies
Why is A the right answer?
Why is A the right answer? Thanks
jingjingxiao11111@gmail.com on November 19, 2021
  • October 2005 LSAT
  • SEC3
  • Q14
2
Replies
Is one word wrong in the correct answer?
Shouldn't the correct answer (A) state it is "less" likely to be misunderstood?
RonMayer on June 7, 2021
  • October 2005 LSAT
  • SEC3
  • Q13
1
Reply
I see why E is correct but why is D wrong?
Can you go over why D doesn't weaken the argument?
capoleway@gmail.com on March 29, 2021
  • October 2005 LSAT
  • SEC3
  • Q24
1
Reply
Why is E correct
I am just struggling with weaken questions
RS1 on December 28, 2020
  • October 2005 LSAT
  • SEC3
  • Q24
1
Reply
explain right answer
Can you please explain why C is incorrect and how to get to the correct answer? thanks
Abigail-Lee on September 13, 2020
  • October 2005 LSAT
  • SEC3
  • Q25
1
Reply
Help please
Why is E wrong
Harper on August 11, 2020
  • October 2005 LSAT
  • SEC3
  • Q11
1
Reply
Answer Choice A
I know that A does not most weaken the argument, but it does still seem to weaken it a little bit...
zgnewquist on January 27, 2020
  • October 2005 LSAT
  • SEC3
  • Q24
1
Reply
Why is B correct?
Why is B correct? Why is D incorrect?
Shiyi-Zhang on January 27, 2020
  • October 2005 LSAT
  • SEC3
  • Q20
1
Reply
Why is C correct?
What evidence did it ignore?
Shiyi-Zhang on January 27, 2020
  • October 2005 LSAT
  • SEC3
  • Q19
1
Reply
Why A over E
Can you please explain why A is the better answer than E please? Thanks!
llowe on January 19, 2020
  • October 2005 LSAT
  • SEC3
  • Q23
1
Reply
Need help!
Why not B or D as answer choice ?
ali on November 5, 2019
  • October 2005 LSAT
  • SEC3
  • Q8
2
Replies
Clarification
A little lost with this one. What is the argument conclusion? What makes answer choice A correct?
tselimovic on January 19, 2015
  • October 2005 LSAT
  • SEC3
  • Q2
1
Reply